Difference between revisions of "2017 AIME I Problems/Problem 3"

m (question and bottom box)
(Solution)
 
(17 intermediate revisions by 10 users not shown)
Line 4: Line 4:
  
 
==Solution==
 
==Solution==
We see that <math>d(n)</math> appears in cycles of <math>20</math>, adding a total of <math>70</math> each cycle.
+
We see that <math>d_n</math> appears in cycles of <math>20</math> and the cycles are <cmath>1,3,6,0,5,1,8,6,5,5,6,8,1,5,0,6,3,1,0,0,</cmath> adding a total of <math>70</math> each cycle.
Since <math>\lfloor\frac{2017}{20}\rfloor=100</math>, we know that by <math>2017</math>, there have been <math>100</math> cycles, or <math>7000</math> has been added. This can be discarded, as we're just looking for the last three digits.
+
Since <math>\left\lfloor\frac{2017}{20}\right\rfloor=100</math>, we know that by <math>2017</math>, there have been <math>100</math> cycles and <math>7000</math> has been added. This can be discarded as we're just looking for the last three digits.
Adding up the first <math>17</math> of the cycle of <math>20</math>, we get that the answer is <math>\boxed{069}</math>.
+
Adding up the first <math>17</math> of the cycle of <math>20</math>, we can see that the answer is <math>\boxed{069}</math>.
 +
~ Maths_Is_Hard
 +
 
 +
==Video Solution==
 +
 
 +
https://youtu.be/BiiKzctXDJg ~ Shrea S
  
 
==See Also==
 
==See Also==
 
{{AIME box|year=2017|n=I|num-b=2|num-a=4}}
 
{{AIME box|year=2017|n=I|num-b=2|num-a=4}}
 
{{MAA Notice}}
 
{{MAA Notice}}

Latest revision as of 23:37, 6 February 2024

Problem 3

For a positive integer $n$, let $d_n$ be the units digit of $1 + 2 + \dots + n$. Find the remainder when \[\sum_{n=1}^{2017} d_n\]is divided by $1000$.

Solution

We see that $d_n$ appears in cycles of $20$ and the cycles are \[1,3,6,0,5,1,8,6,5,5,6,8,1,5,0,6,3,1,0,0,\] adding a total of $70$ each cycle. Since $\left\lfloor\frac{2017}{20}\right\rfloor=100$, we know that by $2017$, there have been $100$ cycles and $7000$ has been added. This can be discarded as we're just looking for the last three digits. Adding up the first $17$ of the cycle of $20$, we can see that the answer is $\boxed{069}$.

~ Maths_Is_Hard

Video Solution

https://youtu.be/BiiKzctXDJg ~ Shrea S

See Also

2017 AIME I (ProblemsAnswer KeyResources)
Preceded by
Problem 2
Followed by
Problem 4
1 2 3 4 5 6 7 8 9 10 11 12 13 14 15
All AIME Problems and Solutions

The problems on this page are copyrighted by the Mathematical Association of America's American Mathematics Competitions. AMC logo.png